1000 CR test D no.1

This topic has expert replies
Master | Next Rank: 500 Posts
Posts: 484
Joined: Sun Jul 30, 2006 7:01 pm
Thanked: 2 times
Followed by:1 members

1000 CR test D no.1

by magical cook » Thu Mar 22, 2007 7:30 pm
Hello,

The answer is A) but I dont seem to get premise/conclusion and therefore can't get it right....


Contrary to the statements of labor leaders, the central economic problem facing America today is not the distribution of wealth. It is productivity. With the productivity of U.S. industry stagnant, or even declining slightly, the economic pie is no longer growing. Labor leaders, of course, point to what they consider an unfair distribution of the slices of pie to justify their demands for further increases in wages and benefits. And in the past, when the pie was still growing, management could afford to acquiesce. No longer. Until productivity resumes its growth, there can be no justification for further increases in the compensation of workers.

Which of the following statements by a labor leader focuses on the logical weakness in the argument above?

(A) Although the economic pie is no longer growing, the portion of the pie allocated to American workers remains unjustly small.
(B) If management fails to accommodate the demands of workers, labor leaders will be forced to call strikes that will cripple the operation of industry.
(C) Although productivity is stagnant, the U.S. population is growing, so that the absolute size of the economic pie continues to grow as well.
(D) As a labor leader, I can be concerned only with the needs of working people, not with the problems faced by management.
(E) The stagnation of U.S. industry has been caused largely by factors—such as foreign competition—beyond the control of American workers.

Senior | Next Rank: 100 Posts
Posts: 55
Joined: Sun Apr 29, 2007 6:25 am
Location: MA
Thanked: 1 times

by bww » Mon May 14, 2007 6:50 pm
Answer here is A.

The premise of the argument is that there is that productivity in the US is stagnant and thusly hurting the labor industries and their labor workers.

B - sure, strikes could be effective in negotiations, but does not address a weakness in the argument and moreover, out of scope.

C - a possible inference, but again, out of scope and also directly contradicts a statement presented as fact in the question stem. not a great choice.

D - true, but too narrow in scope. this isn't about just management. it's about the relationship held by the labor force and management within the economy.

E - could be a valid point, but like B does not address a weakness in the argument and out of scope.

A directly addresses the question stem. It recognizes that the economic pie is no longer growing, but the distribution of productivity (and ultimately rewards and benefits) remains unadjusted to compensate for the efforts of the labor force. It implies that distribution has always been unbalanced (refer to third to last sentence), but at least management could accomodate the labor forces' demands because the markets were growing. Now that the markets have stagnated, the effects of unequal productivity distribution are made much more explicit.